8
$\begingroup$

This question was asked at MSE but never received an answer.

Let $A\subset\mathbb{N}$ be a subset of the natural numbers, and let $\sigma(n)$ denote the sum of divisors of $n$. Recall that we have the bound $\sigma(n) = O(n\log\log n)$. Consider the sums $\sum_{a\in A} 1/a$ and $\sum_{a\in A} 1/\sigma(a)$.

Question. Is there on $A\subset\mathbb{N}$ so that the first sum diverges but the second sum converges ?

If the answer is already known, a reference would more than suffice. Thank you

$\endgroup$

1 Answer 1

11
$\begingroup$

Yes, this is possible. Let $k\ge 3$, say and let $P(k)$ denote the product of the first $k$ primes, and let $A_k$ denote a set of integers that are all multiples of $P(k)$ and with $$ \frac{1}{k \log k} \le \sum_{a\in A_k} \frac{1}{a} \le \frac{2}{k\log k}. $$ Since the harmonic sum diverges, we can clearly choose such $A_k$, and moreover we may arrange for the sets $A_3$, $A_4$, $\ldots$ all to be disjoint (just pick $A_k$ to be the multiples of $P(k)$ in suitable disjoint intervals). Take $A$ to be the union of all the $A_k$ (with $k\ge 3$). Clearly $\sum_{a\in A} 1/a$ diverges.

Now for $a\in A_k$ we have $\sigma(a)/a \ge \sigma(P(k))/P(k) \gg \log k$ by Mertens. Therefore $$ \sum_{a\in A_k} \frac{1}{\sigma(a)} \ll \frac{1}{\log k} \sum_{a\in A_k} \frac 1a \ll \frac{1}{k (\log k)^2}. $$ Therefore $$ \sum_{a\in A} \frac{1}{\sigma(a)} $$ converges.

$\endgroup$
2
  • 2
    $\begingroup$ Nice one (+1): picking $A_k$ as multiples of $P(k)$ and $1$ modulo the $k+1$-th prime, could be also a way too. $\endgroup$ Sep 1, 2015 at 22:46
  • $\begingroup$ @Lucia, nice. How large can you make the asymptotic growth rate of the first sum? I can ask a separate question if you think that's more appropriate. $\endgroup$
    – kodlu
    Jan 14, 2016 at 0:49

Your Answer

By clicking “Post Your Answer”, you agree to our terms of service and acknowledge you have read our privacy policy.

Not the answer you're looking for? Browse other questions tagged or ask your own question.